LSAT and Law School Admissions Forum

Get expert LSAT preparation and law school admissions advice from PowerScore Test Preparation.

General questions relating to LSAT Logical Reasoning.
User avatar
 Quinn Braaten
  • Posts: 3
  • Joined: Aug 02, 2023
|
#103076
Hey, I was doing the StrengthenX question 3 from Chapter 9 of the LRB (the one about navigation system interference in planes), and noticed that (the correct) answer choice E was deemed not to strengthen the argument because "The fact that A and B become popular at the same time does not offer any supporting evidence to the contention that A causes B". However, a couple pages prior, when the methods of strengthening an argument are laid out, one of the methods involves "Showing that when the cause occurs, the effect occurs"

I know I'm misinterpreting one of these guidelines but am unsure of which.

I found a link the the same explanation of the question as is in the bible: viewtopic.php?t=31927

Essentially, howcome answer choice E does not fall into Type B?
 Rachael Wilkenfeld
PowerScore Staff
  • PowerScore Staff
  • Posts: 1358
  • Joined: Dec 15, 2011
|
#103082
Hi Quinn,

Great question! "When the cause occurs, the effect occurs" actually describes the order in which things occur. First the cause, then the effect. Showing that the cause occurs before the effect is what would strengthen, not showing that they both occur at the same time. Causal reasoning is inherently temporal. Answer choice (E) states that they happened simultaneously, which wouldn't support the idea of causal connection.

Hope that helps!
User avatar
 Quinn Braaten
  • Posts: 3
  • Joined: Aug 02, 2023
|
#103086
Thanks that does help! So for that answer choice to have been correct would it have to look something more like "A (electronic devices) became widely popular just before the time that B (systems interference) began to occur"?
User avatar
 Jeff Wren
PowerScore Staff
  • PowerScore Staff
  • Posts: 390
  • Joined: Oct 19, 2022
|
#103091
Hi Quinn,

First, I want to make one additional point regarding your initial question. You wrote, ""The fact that A and B become popular at the same time does not offer any supporting evidence to the contention that A causes B".

I think that you may have missed an important distinction in Answer E and in the explanation that was given.

First, let's be clear about what is the proposed cause and effect in the argument. The cause is the passenger electronic devices (combined with the modern navigation system's low power circuitry) and the effect is the interference.

Answer E states that the passenger electronic devices and the low power circuitry occurred about the same time. This is not showing the cause and the effect together. The effect is the interference. For E to show "when the cause occurs, the effect occurs" it would need to state that the interference started happening soon after the electronic devices became popular (or something to that effect). For all we know based on Answer E, the interference problems may have started much later than the introduction of the passenger electronic devices and in fact be due to something completely unrelated.

In the explanation that you quote, you have "B" as being the same in the effect as in the first part (the correlation), which is not the case. The actual explanation distinguishes that the fact that the circuitry and electronic devices becoming popular does not provide support for electronic devices causing the interference.

In your second question, you correctly identify B (the effect) as the interference, so that would strengthen the causal argument.

Hope that helps!

Get the most out of your LSAT Prep Plus subscription.

Analyze and track your performance with our Testing and Analytics Package.